Doubt regarding derivation of De Broglie relation

In summary, the author uses symmetry considerations and vector expressions to show that the integration constant c in the de Broglie relation must be zero. This implies that the mean momentum is equal to the mean wavenumber multiplied by hbar. When the mean momentum is zero, the wavepacket will spread out in time due to the various frequency components having different velocities. The group velocity of a wave is approximately equal to the derivative of frequency with respect to wavenumber, and to find the velocity of a group packet, one would need to evaluate this derivative at various points in space and then sum average the values.
  • #1
weezy
92
5
For someone who's familiar with the de Broglie relation it's easy to say that for k=0 we have p=0 but how would we know that before deriving the result? In this image the author derives de Broglie relation by considering a wave packet in motion. As you can see where I have star-marked the author sets integration constant c=0 without giving reason. This has been bothering me since you can't just
8PT69.jpg
 
Physics news on Phys.org
  • #2
I think you can use symmetry considerations to understand that the integration constant [itex]c[/itex] must be zero.

In particular, we would require that changing the sign (direction) of [itex]p[/itex] also changes the sign of [itex]k[/itex]. This is perfectly legitimate, as this is what a coordinate flip would do.

If
[itex]p=\hbar k + c,[/itex]
and
[itex](-p) = \hbar (-k) +c[/itex]
then adding these two equations together gives
[itex]0=2c[/itex], or [itex]c=0[/itex].
 
  • #3
jfizzix said:
I think you can use symmetry considerations to understand that the integration constant [itex]c[/itex] must be zero.

In particular, we would require that changing the sign (direction) of [itex]p[/itex] also changes the sign of [itex]k[/itex]. This is perfectly legitimate, as this is what a coordinate flip would do.

If
[itex]p=\hbar k + c,[/itex]
and
[itex](-p) = \hbar (-k) +c[/itex]
then adding these two equations together gives
[itex]0=2c[/itex], or [itex]c=0[/itex].
I think where this proof could be wrong is because k is a scalar quantity isn't it? And scalar quantities can't flip signs like that. Correct me if I'm wrong.
 
  • #4
the wavenumber [itex]k[/itex] is a vector quantity with magnitude and direction, just like momentum. If you take all the above quantities to refer to one-dimensional propagation, this will work out just fine.

Alternatively, if you explicitly use the vector expressions of [itex]k[/itex] and [itex]p[/itex] throughout this derivation, you should be able to show that
[itex]\hbar = \frac{\partial p_{x}}{\partial k_{x}}=\frac{\partial p_{y}}{\partial k_{y}}=\frac{\partial p_{z}}{\partial k_{z}}[/itex].
It gets a little tricky toward the end, but since what you derive must be true for all [itex](p_{x},p_{y},p_{z})[/itex], you can make some simplifications.
 
Last edited:
  • #5
jfizzix said:
the wavenumber [itex]k[/itex] is a vector quantity with magnitude and direction, just like momentum. If you take all the above quantities to refer to one-dimensional propagation, this will work out just fine.

Alternatively, if you explicitly use the vector expressions of [itex]k[/itex] and [itex]p[/itex] throughout this derivation, you should be able to show that
[itex]\hbar = \frac{\partial p_{x}}{\partial k_{x}}=\frac{\partial p_{y}}{\partial k_{y}}=\frac{\partial p_{z}}{\partial k_{z}}[/itex].
It gets a little tricky toward the end, but since what you derive must be true for all [itex](p_{x},p_{y},p_{z})[/itex], you can make some simplifications.
While I find your proof correct now(after considering k as a vector) and satisfying what's the physical reason you would give to argue the constant must be 0?
 
  • #6
When I visualizer the situation for P=0, the wavelength becomes infinite. What does this imply? @jfizzix
 
  • #7
weezy said:
While I find your proof correct now(after considering k as a vector) and satisfying what's the physical reason you would give to argue the constant must be 0?

The physical reason I would give is that the magnitude of the wave number as well as the momentum should be constant under a reflection of coordinates.
 
  • #8
weezy said:
When I visualizer the situation for P=0, the wavelength becomes infinite. What does this imply? @jfizzix
It helps to recall that the group velocity of a wave is only approximately [itex]\frac{\partial\omega}{\partial k}[/itex], and that each frequency component travels at its own velocity, so that the group velocity is a statistical average (equal to the rate of change of the mean position of the pulse).

The derivation we're discussing here shows that the mean momentum is equal to hbar times the mean wavenumber. If the mean momentum is zero, the mean position of the pulse is stationary, but the pulse itself spreads out in time, since it will have components moving at nonzero velocities.

If the wave's momentum components were tightly clustered near zero, then the wavepacket would spread out in opposite directions very quickly.
 
  • Like
Likes weezy
  • #9
jfizzix said:
It helps to recall that the group velocity of a wave is only approximately [itex]\frac{\partial\omega}{\partial k}[/itex], and that each frequency component travels at its own velocity, so that the group velocity is a statistical average (equal to the rate of change of the mean position of the pulse).

The derivation we're discussing here shows that the mean momentum is equal to hbar times the mean wavenumber. If the mean momentum is zero, the mean position of the pulse is stationary, but the pulse itself spreads out in time, since it will have components moving at nonzero velocities.

If the wave's momentum components were tightly clustered near zero, then the wavepacket would spread out in opposite directions very quickly.
You mean the phase component waves move out rapidly in opp. directions?
 
  • #10
I mean the positive frequency components move in the opposite direction as the negative frequency components.

If the pulse is very narrow in space/time, then it will have to have very many wavenumber/frequency components, some of which will be moving a lot faster, simply because there are more significant frequency components making up the wave.
 
  • Like
Likes weezy
  • #11
jfizzix said:
It helps to recall that the group velocity of a wave is only approximately [itex]\frac{\partial\omega}{\partial k}[/itex], and that each frequency component travels at its own velocity, so that the group velocity is a statistical average (equal to the rate of change of the mean position of the pulse).

The derivation we're discussing here shows that the mean momentum is equal to hbar times the mean wavenumber. If the mean momentum is zero, the mean position of the pulse is stationary, but the pulse itself spreads out in time, since it will have components moving at nonzero velocities.

If the wave's momentum components were tightly clustered near zero, then the wavepacket would spread out in opposite directions very quickly.
Also when you say that group velocity is [itex]\frac{\partial\omega}{\partial k}[/itex] does this mean that if I were to find the velocity of a group packet, I'd have to evaluate this derivative at various points in space? And then sum average all the values to get an approximate number? Or just evaluate the derivatives for all values of k in the group and then sum average over k?
 
  • #12
Where a wave [itex]\Psi(x,t)[/itex]can be decomposed into an integral of plane waves, each with amplitude [itex]A(k)[/itex], we have:
[itex]\Psi(x,t)=\frac{1}{\sqrt{2\pi}}\int dk A(k) e^{i(kx - \omega(k) t)}[/itex].
The time derivative of the mean position [itex]\langle x\rangle[/itex] as weighted according to intensity [itex]|\Psi(x,t)|^{2}[/itex], can be given by the following formula.

[itex]\frac{d\langle x\rangle}{dt}=\int dk |A(k)|^{2} \Big(\frac{\partial \omega(k)}{\partial k}\Big)[/itex],

so that when [itex]\Big(\frac{\partial \omega(k)}{\partial k}\Big)[/itex] is nearly constant over the significant wavenumbers of [itex]|A(k)|^{2}[/itex], we get the approximate formula for the group velocity.

Then, it is understood that [itex]\frac{\partial \omega(k)}{\partial k}[/itex] is to be evaluated at the peak value of [itex]k[/itex] for the pulse under consideration. The three-dimensional version of this derivation follows the same steps without a substantial increase in difficulty.

Technically, the group velocity is the velocity of the pulse envelope (which itself only makes sense for very narrowband pulses), but this formula is how I make sense of it.
 
  • Like
Likes weezy
  • #13
Thanks for your time and answers :)
 

Related to Doubt regarding derivation of De Broglie relation

1. How was the De Broglie relation derived?

The De Broglie relation was derived by Louis de Broglie in 1924 through a thought experiment involving the concept of wave-particle duality. He proposed that just as light can exhibit both wave and particle properties, so can matter. By applying the equations of Einstein's theory of special relativity to a particle with mass, de Broglie was able to derive the relationship between a particle's wavelength and its momentum, which is now known as the De Broglie relation.

2. What is the significance of the De Broglie relation?

The De Broglie relation is significant because it provides a link between the wave and particle properties of matter, bridging the gap between classical mechanics and quantum mechanics. This relation also helps to explain the behavior of particles at the atomic and subatomic level, and has been confirmed through various experiments, including the famous Davisson-Germer experiment which demonstrated the wave-like nature of electrons.

3. Can the De Broglie relation be applied to macroscopic objects?

No, the De Broglie relation is only applicable to particles with very small mass, such as electrons. This is because the wavelength of a macroscopic object would be so small that it would be undetectable and have no practical significance.

4. How does the De Broglie relation relate to Heisenberg's uncertainty principle?

The De Broglie relation is one of the founding principles of quantum mechanics and is closely related to Heisenberg's uncertainty principle. This principle states that it is impossible to know both the exact position and momentum of a particle at the same time. This is because the more accurately we measure one property, the less accurately we can measure the other property. The De Broglie relation provides a mathematical explanation for this uncertainty by showing that the momentum of a particle is inversely proportional to its wavelength.

5. Can the De Broglie relation be used to predict the behavior of particles?

Yes, the De Broglie relation can be used to predict the behavior of particles, particularly at the atomic and subatomic level. It has been successfully used in various experiments and is an important tool in understanding the quantum world. However, it is important to note that the De Broglie relation is a probabilistic relationship and cannot predict the exact position or momentum of a particle, but rather the likelihood of these properties within a certain range.

Similar threads

  • Quantum Interpretations and Foundations
Replies
6
Views
2K
  • Quantum Interpretations and Foundations
Replies
4
Views
1K
  • Quantum Interpretations and Foundations
Replies
8
Views
1K
  • Quantum Interpretations and Foundations
2
Replies
40
Views
5K
  • Quantum Interpretations and Foundations
Replies
17
Views
3K
  • Quantum Interpretations and Foundations
Replies
14
Views
4K
  • Other Physics Topics
Replies
5
Views
944
  • Quantum Interpretations and Foundations
Replies
1
Views
2K
  • Quantum Interpretations and Foundations
Replies
31
Views
4K
Back
Top